Übergangs-Marathon Mathematik - Seite 4

Neue Frage »

Scherzbold Auf diesen Beitrag antworten »

.
tmo Auf diesen Beitrag antworten »

Wenn du aus dem Unendlichzeichen eine Sechs machst, kannst du sogar den "Scherz" weglassen smile
Scherzbold Auf diesen Beitrag antworten »

Kann er, muss er aber nicht. LOL Hammer Du kannst es auch, wenn du nur fest an dich glaubst. Wink

Stimmt tmo. Augenzwinkern
Scherzbold Auf diesen Beitrag antworten »

Der erste Satz bezog sich auf einen nun gelöschten Kommentar.
Nofeykx Auf diesen Beitrag antworten »

Vielleicht nicht sehr schwer, aber eine schöne Aufgabe.

Sei und für alle .

Ist konvergent?
HAL 9000 Auf diesen Beitrag antworten »

Ich löse mal auf, da ich gern selbst eine Aufgabe einstellen will. smile

Die Reihe divergiert bestimmt gegen - Begründung:

Offenbar sind alle Reihenglieder positiv, damit ist die Partialsummenfolge eine streng monoton wachsende Folge positiver Zahlen. Angenommen, ist beschränkt, dann ist es wegen der Monotonie auch konvergent gegen eine Zahl , für die zum einen für alle gilt, zum anderen gibt es für alle ein mit für alle .

Betrachten wir speziell , so gilt für das zugehörige die Ungleichung und gemäß Rekursion dann im Widerspruch zu .

Daher war die Annahme falsch, d.h., ist unbeschränkt und damit bleibt aufgrund der Monotonie nur noch die bestimmte Divergenz gegen .



Neue Aufgabe (angeregt durch dieses Problem):

Zitat:
Aufgabe 51

Für alle positiven ganzen Zahlen gilt

Auch wenn ich den Weg nicht vorschreiben will: Ein Beweisweg ohne Vollständige Induktion ist möglich. Augenzwinkern
 
 
RavenOnJ Auf diesen Beitrag antworten »

Ich vergaß, die Lösung gehört ja hier rein, nicht in die Diskussionsseite.

Da anscheinend von anderer Seite nichts mehr kommt, eine hoffentlich nicht ganz unklare Lösung für 51. Kurz ist allerdings was anderes Augenzwinkern . Interessant wäre es, wie eine kurze, elegante Lösung für das Problem aussieht (@HAL, @IfindU).

Es gilt: , sowie .

Man kann nun schreiben
.

Der Beweis erfolgt über vollständige Induktion mit der Induktionsvoraussetzung

.

Es ist nun zu zeigen, dass gilt


Dabei sind die Fälle A) n ungerade und B) n gerade zu unterscheiden. Im Fall B) lässt sich der erste Summand und die 1 rechts vom Gleichheitszeichen unter die Summe ziehen und damit die Summe erweitern, im Fall A) heben diese beiden Größen sich auf. Dort ist äquvalent



Im Fall B) ist , die Summe lässt sich also zu (**) erweitern.

Im Fall A) ist die letzte Gleichung in (**) klar wegen und .

Im Fall B) gilt


wobei in der ersten Summe der letzten Gleichung eine Indexverschiebung vorgenommen und in beide Summen eine 1 integriert wurde. Den Induktionsanfang n=1 schenke ich mir, da er „trivial” ist. q.e.d.

Vielleicht hat der @Drummer eine neue Aufgabe !?
IfindU Auf diesen Beitrag antworten »

Entschuldige, ich war beschäftigt und wollte dann gucken, ob DrummerS sich nocht meldet.

Meine Lösung war:
.

Der Integrand lässt sich nach binomischen Lehrsatz nun schreiben als
. Also
,
wobei man im letzten Schritt benutzt hat, dass es die geometrische Summenformel war.
Also weiter
. Auswerten und shiften liefert damit .

Wenn man alles so ausfuehrlich schreibt, wirkt es gar nicht mehr so elegant Big Laugh
RavenOnJ Auf diesen Beitrag antworten »

Na ja, kürzer als meins auf alle Fälle. Eleganter wohl auch. Freude Auf die Idee mit dem Abstecher in die Analysis bin ich leider nicht gekommen. traurig
IfindU Auf diesen Beitrag antworten »

@RavenOnJ Da ich in der Analysis Zuhause bin, war fuer mich kein Abstecher noetig.

Ich weiß nicht wie interessant die Aufgabe ist, aber besser als nichts (man darf sie gerne mit einer interessanteren Aufgabe übergehen)

Zitat:
Für welche existiert das Integrale und für welche das Integral , wobei die Euler-Konstante ist?
HAL 9000 Auf diesen Beitrag antworten »

Vielleicht bieten wir mehrere Aufgaben zur Wahl, damit es nicht immer zu solchen Threadstagnationen kommt ... Ist natürlich nur eine Ausrede von mir, da mir gerade folgende interessante Aufgabe in die Hände gefallen ist, die ich hier gern darlegen möchte: Augenzwinkern

Zitat:
Man untersuche, für welche positiven ganzen Zahlen es eine Fibonacci-Zahl gibt, deren Dezimaldarstellung auf mindestens Neunen endet.
Huggy Auf diesen Beitrag antworten »

Zitat:
Lösung zu Nr. 53 (?), der letzten Aufgabe von HAL

Zu jedem n gibt es eine Fibonacci-Zahl, deren Dezimaldarstellung auf mindestens n Neunen endet.

Da die Aufgabe erst kurz im Board steht, halte ich meinen Beweis noch etwas zurück.
HAL 9000 Auf diesen Beitrag antworten »

Dann gibt's gleich noch eine Zusatzaufgabe: Man gebe einen Algorithmus an, der für eine beliebige vorgegebene -stellige Ziffernfolge prüft, ob es eine Fibonacci-Zahl mit genau diesen Endziffern gibt. (Er sollte nach endlicher Zeit terminieren, Effizienz sei mal außen vor gelassen.)
Huggy Auf diesen Beitrag antworten »

Mein Beweis liefert auch dafür eine Lösung.
HAL 9000 Auf diesen Beitrag antworten »

Hab ich mir gedacht. Big Laugh
Fragen über Fragen Auf diesen Beitrag antworten »

Da das vor rund 15 Jahren mal ne Matheolympiaden-Aufgabe war und ich süchtig nach solchen Aufgaben bin, möchte ich mich auch mal wieder zu Wort melden Big Laugh .

Vorabbemerkung: Das Latex hier ist, nunja, eigenwillig. Man darf eine Folge nicht b_n nennen, sonst gibts Gemecker. Umbenennen in d_n gibt Linderung, allerdings darf man Kongruenzgleichungen nur mit b_n, nicht mit d_n formulieren. p als Index ist auch schlecht, vielleicht will Latex manchmal Pi daraus machen Big Laugh . Entsprechende Problemstellen habe ich daher stattdessen in eine Mathjax-Umgebung eingefügt.

Zitat:
Die Fibonaccifolge sei gegeben durch für alle .
Gegeben sei ein natürliches . Wir betrachten die Fibonaccifolge modulo , also die Folge , die gegeben sei durch und für alle .

Damit gilt auch für alle .

Lemma 1: Die Folge ist ab einem gewissen Index periodisch mit Periodenlänge kleiner gleich .

Beweis: Wenn es zwei Indizes gibt, sodass gilt, so gilt auch , und induktiv folgt für alle , wobei gilt.

Nun gibt es genau verschiedene Tupel mit . Nach dem Schubfachprinzip gibt es zwei Indizes mit . Nach Obigem bedeutet das aber gerade, dass die Folge ab dem Index periodisch ist und für die Periodenlänge gilt . Das beendet den Beweis.


Lemma 2: Falls die Folge ab einem gewissen minimalen Index periodisch ist, so gilt , d.h. es gibt keine Vorperiode.

Beweis: Für folgt die Aussage. Sei also und es gebe ein natürliches , sodass für alle .

Es gilt insbesondere und
Wir haben .
Das heißt es gilt auch , im Widerspruch zur Minimalität von . Das beendet den Beweis.

Lemma 1 und Lemma 2 liefern, dass ein exisitert, sodass für alle gilt.

Betrachte die Folge , mit für alle .

Lemma 1 und Lemma 2 gelten ebenfalls für die Folge , da und für alle gilt, und nur diese beiden Eigenschaften wurden für den Beweis der beiden Lemmata benötigt.
Somit existiert ein , sodass gilt. Offensichtlich gilt stets , und somit gilt , also . Daraus folgt, dass auf Neunen endet. Da beliebig war, schließen wir, dass für jedes eine Fibonaccizahl existiert, die auf Neunen endet.

Zur Zusatzaufgabe:
Übersetze die gegebene -stellige Ziffernfolge in eine natürliche Zahl (indem eventuelle führende Nullen gestrichen werden). Untersuche die ersten Glieder der Folge . Genau dann wenn die Zahl dort als Folgeglied auftaucht, endet eine Fibonaccizahl mit der gegebenen Ziffernfolge.
HAL 9000 Auf diesen Beitrag antworten »

Kann man so akzeptieren, obwohl ich den letzten Teil mit der Einführung der Folge reichlich nebulös dargestellt empfinde: Die Verbindung zur Folge bzw. ist m.E. sehr verwickelt ausgeführt. Viel einfacher geht es doch so: Man setzt die Fibonacci-Folge in negativer Indexrichtung fort, d.h. via . Dann ist und und mit Hilfe deiner nachgewiesenen Periodizität dann , was zu beweisen war.

Zitat:
Original von Fragen über Fragen
Übersetze die gegebene -stellige Ziffernfolge in eine natürliche Zahl (indem eventuelle führende Nullen gestrichen werden). Untersuche die ersten Glieder der Folge . Genau dann wenn die Zahl dort als Folgeglied auftaucht, endet eine Fibonaccizahl mit der gegebenen Ziffernfolge.

Ja, ich hatte ja gesagt, dass ich auf Effizienz verzichte. Big Laugh

Ein "klein wenig" schneller geht es, da man bereits bei ersten Auftreten der Sequenz die Periode festgestellt hat. Macht durchaus einen Unterschied: Bei etwa ist , das ist schon ein wenig kleiner als . Augenzwinkern
Huggy Auf diesen Beitrag antworten »

Zitat:
Original von HAL 9000
Die Verbindung zur Folge bzw. ist m.E. sehr verwickelt ausgeführt.

Mein Beweis war da auch kürzer formuliert.
Nachdem die Periodizität der Endziffern klar war, habe ich zunächst mal die ersten tatsächlichen Perioden gesucht. Es ergab sich:

k = 1, Periode 60
k = 2, Periode 300
k = 3, Periode 1500

Die Endziffern 9, 99, 999 fand ich jedes mal 2 Indizes vor Beginn der neuen Periode, wobei die Endziffer 9 in der ersten Periode noch bei anderen Indizes auftritt. Danach war es leicht, mit der Rekursion für die Endziffernfolgen zu zeigen, dass dies allgemeingültig ist.
Huggy Auf diesen Beitrag antworten »

Zitat:
Aufgabe 54: Eine merkwürdige Folge von Integralen

Es sei



Angeregt durch einen Zeitschriftenartikel über experimentelle Mathematik ließ mal jemand diese Integrale von einem Programm numerisch berechnen. Das Programm galt auf diesem Gebiet als zuverlässig. Es ergab sich:







Der Zahlenwert kommt einem bekannt vor und das Schema ist klar. Es gibt keinen Grund mit dem Experiment fortzufahren. Der "jemand" war jedoch hartnäckig und machte weiter. Es ergab sich immer wieder



bis . Doch dann zu seiner Verblüffung:



Was war die Ursache?
Fragen über Fragen Auf diesen Beitrag antworten »

Zitat:
Original von HAL 9000
Viel einfacher geht es doch so: Man setzt die Fibonacci-Folge in negativer Indexrichtung fort, d.h. via .

Bei mir passiert ja das gleiche: anstatt die Fibonacci-Folge in negativer Indexrichtung fortzusetzen, setze ich die Fibonacci-Folge modulo in negativer Indexrichtung fort, und man sieht dem Glied sofort an, dass es die gewünschte Zahl 9...9 ist. Natürlich wäre es übersichtlicher gewesen, wenn ich dafür nicht eine neue Folge eingeführt hätte, sondern in negative Indexrichtung fortgesetzt hätte, daran hatte ich nicht gedacht.


Zitat:

Ein "klein wenig" schneller geht es, da man bereits bei ersten Auftreten der Sequenz die Periode festgestellt hat.

Freilich Augenzwinkern .
Leopold Auf diesen Beitrag antworten »

zu Aufgabe 54

Natürlich ein Fehler in der CPU. Im Ernst - was es nicht alles gibt!

Mit Hilfe von rechnet man:



Ein Faktor mehr und mit derselben Formel von oben umgeformt:



Iterativ erkennt man: Bei jedem Schritt verdoppelt sich die Anzahl der Summanden, der Vorfaktor halbiert sich. Daher gilt:



mit geeigneten Koeffizienten . Jedes ist von der Gestalt



Jede Kombination von Plus- und Minuszeichen ist zulässig. Bei Operationen und jeweils zwei Möglichkeiten (plus oder minus) erhält man so die möglichen Koeffizienten.
Den dem Werte nach kleinstmöglichen Koeffizienten bekommt man, wenn man immer das Minuszeichen nimmt:



Nun ist





Für sind somit alle Koeffizienten . Für gilt aber stets



wie man mit Hilfe der Substitution und dem bekannten Wert sehen kann. Für gilt daher:



Ist dagegen , so gilt:

Im Falle ist der kleinste Koeffizient, wie oben berechnet, kleiner 0, der nächstgrößere



ist aber bereits . Also ist nur einer der Koeffizienten negativ. Somit gilt:



Ein schönes Beispiel für eine mißlingende unvollständige Induktion.
HAL 9000 Auf diesen Beitrag antworten »

Definiert man für beliebige reelle Zahlen den Integralwert

,

so gilt übrigens die Rekursion



,

letzteres basiert auf dem oben schon von Leopold angegeben Additionstheorem. Im vorliegenden Fall wird der Spezialfall betrachtet.

Für positive sowie bedeutet das , "knapp" unterhalb (genauer gesagt sowie monoton falllenden ) jedoch .
Huggy Auf diesen Beitrag antworten »

@Leopold und HAL

Schnell und perfekt gelöst!
HAL 9000 Auf diesen Beitrag antworten »

Zitat:
Original von Leopold
Ein schönes Beispiel für eine mißlingende unvollständige Induktion.

Und auch ein Beispiel, wie CAS mitunter grandios versagen können: MuPad liefert mir bereits bei dem noch moderaten int(sin(4*x)/x*cos(x)*cos(x/2),x=0..infinity) die lapidare Antwort undefined. geschockt
Huggy Auf diesen Beitrag antworten »

Mathematica löst die Integrale bis exakt. Weiter konnte ich nicht gehen, da die Zeiten auf meinem alten lahmen Rechner dann zu lang werden.
Fragen über Fragen Auf diesen Beitrag antworten »

Mal was Kurzes, aber Hübsches:
Zitat:
Aufgabe 55: Zeige, dass wegzusammenhängend ist.
Huggy Auf diesen Beitrag antworten »

Zwei Punkte kann man doch immer simpel durch einen Streckenzug verbinden, der aus maximal 3 Strecken besteht, die alle waagrecht oder senkrecht verlaufen und die in liegen, weil man eine ihrer Koordinaten jeweils irrational wählen kann.

Verstehe ich die Aufgabe irgendwie falsch?
Fragen über Fragen Auf diesen Beitrag antworten »

Deine horizontalen und vertikalen Streckenzüge müssten das Bild eines geschlossenen Intervalls unter einer stetigen Funktion sein. Das ist nicht möglich: So eine horizontale/vertikale Strecke ist homöomorph zu mit einem geschlossenen Intervall , dessen Randpunkte irrational sind. Allerdings ist nicht zusammenhängend, also auch nicht wegzusammenhängend.

Nebenbei verstehe ich dann auch nicht, wieso du manchmal drei Strecken brauchst - wenn horizontale und vertikale Strecken klappen würden, so bräuchtest du doch bloß zwei.

PS: Ihr könnt auch die allgemeinere Aussage beweisen: Ist abzählbar, so ist wegzusammenhängend.
Fragen über Fragen Auf diesen Beitrag antworten »

Nachtrag: Sorry, ich habe übersehen, dass ja auch Punkte möglich sind, bei denen lediglich eine Koordinate irrational ist. 10001000Nick1 hat mir per PN eine richtige Lösung geschickt, die sich mit dem deckt, was Huggy angedeutet hat.
Das kommt davon, dass ich nur den allgemeinen Fall gelöst hatte und mir über die ursprüngliche Aufgabe gar keine Gedanken gemacht hab ^^
10001000Nick1 Auf diesen Beitrag antworten »

Also dann hier erstmal meine Lösung zu der ursprünglichen Aufgabe. Ist vielleicht nicht der eleganteste Weg, dafür aber (wie ich meine) der einfachste. Augenzwinkern

Seien . sei irgendeine irrationale Zahl.

1. Fall: .

Dann ist mit

und



ein stetiger Weg mit und .

2. Fall: .
Funktioniert ähnlich.

3. Fall: .

,

und



4. Fall: .
Ähnlich wie 3.

Zu der Bemerkung, dass es immer mit zwei Streckenzügen geht: Falls , geht es mit zwei horizontalen oder vertikalen Streckenzügen nicht.
10001000Nick1 Auf diesen Beitrag antworten »

Und hier für die allgemeinere Aussage:

Seien .
sei eine Gerade, auf der weder noch liegen und die die Verbindungsstrecke zwischen und schneidet.

Für jeden Punkt auf bezeichnen wir mit den Streckenzug von über nach .

Für zwei verschiedene schneiden sich die Streckenzüge und nur in den beiden Punkten .
Weil es überabzählbar viele Punkte auf gibt, und wegen der Abzählbarkeit von , muss es dann einen solchen Streckenzug geben, auf dem keiner der Punkte aus liegt.
Fragen über Fragen Auf diesen Beitrag antworten »

Das passt. Freude
Huggy Auf diesen Beitrag antworten »

Zitat:
Aufgabe 56

Mal etwas für Freunde der Geometrie. Gegeben sei eine Gerade g und ein nicht auf ihr liegender Punkt P. Von dem Punkt P soll das Lot auf die Gerade g gefällt werden.

Mit den Hilfsmitteln Zirkel und Lineal ist das eine der ersten Schulaufgaben in synthetischer Geometrie. Das Hilfsmittel Zirkel soll nun nicht zur Verfügung stehen, Lineal aber schon. Stattdessen gebe es das Hilfsmittel Rechter Winkel. Dieses Hilfsmittel gestattet es, auf einer Geraden g in einem auf ihr liegenden Punkt X die Senkrechte zu errichten.
riwe Auf diesen Beitrag antworten »

mir gefällt das natürlich sehr !
aber ich halte mich noch etwas zurück Augenzwinkern
Leopold Auf diesen Beitrag antworten »

Wenn man sich so an den Zirkel gewöhnt hat, fällt es einem schon schwer, sich zu beschränken ...
Ich habe es im wesentlichen mit Hilfe von Spiegelungen bewerkstelligt.
riwe Auf diesen Beitrag antworten »

ein Bilderl von mir Augenzwinkern
Leopold Auf diesen Beitrag antworten »

Ich verstehe deine Konstruktion nicht. Die Gerade scheint bei dir zu heißen. In zwei Punkten wird das Lot errichtet. Aber wie wird konstruiert? Wo geht überhaupt in die Konstruktion ein?
riwe Auf diesen Beitrag antworten »

eentschuldige, das war mist unglücklich
Leopold Auf diesen Beitrag antworten »

eine Lösung zur Nr. 56
[attach]42030[/attach]
riwe Auf diesen Beitrag antworten »

und zur Ergänzung meine nicht ganz so elegante Lösung:

die Aufgabe zerfällt in 3 Teile:
1) konstruiere auf der Geraden a mit Hilfe des RechteWinkelMaßes (RWM ) eine
beliebige Strecke AC samt Mittelpunkt M

dazu wählt man 2 beiliebige Punkte A und B auf a,
senkrechte Gerade n zu a mit RWM und beliebige gerade g mit dem Lineal L durch B,
Schnittpunkt mit n sei P1, mit RWM Parallele zu a, dann 2 mal "Zickzack"
mit RWM zu g usw...ergibt M und C

damit hat man Ziel 1) erreicht.

2) man konstruiere ein Parallele zu a durch P

Gerade s durch A und P, wähle auf s einen "geeigneten" Punkt S.
verbinde S mit M und C und C mit P, so erhält man den Schnittpunkt M1.
die Gerade durch A und M1 schneidet SC in Q.

PQ ist die gesuchte Parallele zu a durch P
den Beweis dazu liefern Ceva und die Umkehrung des Strahlensatzes

3) RWM auf P anwenden
Neue Frage »
Antworten »



Verwandte Themen

Die Beliebtesten »
Die Größten »
Die Neuesten »